IMO Shortlist 1967 problem 3


Kvaliteta:
  Avg: 3,0
Težina:
  Avg: 3,3
Dodao/la: arhiva
2. travnja 2012.
LaTeX PDF
Prove that for arbitrary positive numbers the following inequality holds
\frac{1}{a} + \frac{1}{b} + \frac{1}{c} \leq \frac{a^8 + b^8 + c^8}{a^3b^3c^3}.
Izvor: Međunarodna matematička olimpijada, shortlist 1967